6
$\begingroup$

M a finitely generated module over a commutative ring A. I can't think of an example of two maximal linearly independent subsets of M having different cardinality. I know that they all have the same cardinality if A is integral domain. Any suggestions are welcome!

$\endgroup$
1
  • 1
    $\begingroup$ kwan, I believe that this question is based upon a misunderstanding of Georges' answer here mathoverflow.net/questions/29993/rank-of-a-module. When A is a domain, yes, the quantity you're talking about is well-defined and equal to rank. See Bruns and Herzog Section 1.4 for more in this case. When A is not a domain, this is not a very interesting quantity, since it does not obey the usual rules from linear algebra. E.g. let R=k[x,y]/(xy). Then R/(x) and R/(y) contain no lin.indep. elements (each is killed by an element of R), but R/(x)\oplus R/(y) contains (1,1), which is lin.indep. $\endgroup$ Jun 30, 2010 at 17:39

2 Answers 2

10
$\begingroup$

I found an old paper by Lazarus (Les familles libres maximales d'un module ont-elles le meme cardinal?, Pub. Sem. Math. Rennes 4 (1973), 1-12) which contains the the following result: Let A be a commutative ring with unit and M an A-module. In the following situations, maximal linearly independent subsets of M have the same cardinality:

  1. If M is a free A-module of infinite rank.

  2. If A is reduced and has only finitely many minimal primes (e.g. integral domain, reduced Noetherian ring)

  3. If A is Noetherian and M is a free A-module.

  4. If A is Noetherian and M is a submodule of a free A-module of finite rank.

  5. If A is Noetherian and M has an infinite linearly independent subset.

  6. If A is Noetherian and M is a submodule of a flat A-module.

  7. If A is Artin local and the zero ideal $(0)\subset A$ is irreducible.

And the examples given in the paper of modules not satisfying this same cardinality property are highly nontrivial.

$\endgroup$
8
  • $\begingroup$ Dear kwan: Here is how I understand your answer. Lazarus proves the existence of non-equipotent maximal linearly independent subsets of modules over commutative rings. Is this interpretation of your wording correct? $\endgroup$ Jul 3, 2010 at 16:11
  • 1
    $\begingroup$ Dear Pierre-Yves, he actually constructs two non-equipotent maximal linearly independent subsets of a module M over a commutative ring A which do not satisfy the conditions listed in the answer. $\endgroup$
    – ashpool
    Jul 4, 2010 at 0:11
  • 4
    $\begingroup$ Dear Victor, he gives one example of a module over a Noetherian ring which has maximal linearly independent subsets $S_{1}$ and $S_{2}$ where $|S_{1}|=1$ and $|S_{2}|=2$. $\endgroup$
    – ashpool
    Jul 4, 2010 at 0:17
  • 1
    $\begingroup$ This is very interesting! I wasn't able to find the paper online, would you mind adding this example to your answer? $\endgroup$ Jul 4, 2010 at 1:39
  • 3
    $\begingroup$ Here is the example: $A=k[x,y]/(x,y)^{2}$. Then the $A$-module $A^{3}/((\bar{x},0,\bar{x}),(\bar{y},\bar{x},0))$ has a maximal linearly independent singleton { (1,0,0) } and a linearly independent subset { (0,1,0), (0,0,1) }. $\endgroup$
    – ashpool
    Jul 4, 2010 at 15:39
1
$\begingroup$

If you consider rings that are not necessarily commutative, here's an example: let $V$ be a countable dimensional vector space over a field $F$, and let $A$ be the ring of all endomorphisms of $A$. I claim that $A\cong A\oplus A$ (as left $A$-modules); if so, then using (and iterating) this isomorphism you can find maximal linearly independent subsets of any finite cardinality.

To see that $A\cong A\oplus A$, it suffices to exhibit a two-element $A$-basis for $A$. Let $e_1,e_2,\ldots$ be a basis for $V$. Let $f_1\in A$ be the endomorphism that maps $e_2,e_4,e_6,\ldots$ to $e_1,e_2,e_3,\ldots$, respectively, and maps every odd-indexed basis element to $0$; let $f_2\in A$ be the endomorphism that maps $e_1,e_3,e_5,\ldots$ to $e_1,e_2,e_3,\ldots$, and maps the even-indexed basis elements to $0$. Then $f_1,f_2$ spans $A$: if $\varphi \in A$, then we can write $\varphi$ as $\varphi=gf_1+hf_2$, where $g(e_i)=\varphi(e_{2i})$ and $h(e_j)=\varphi(e_{2j-1})$. To see that $f_1$ and $f_2$ are $A$-linearly independent, suppose that $af_1+bf_2=0$; evaluating at the odd indexed $e_i$ shows that $b(e_j)=0$ for all $j$, and evaluating at the even indexed $e_i$ shows $a(e_j)=0$ for all $j$. Thus, $f_1,f_2$ is also a basis for $A$, which gives an isomorphism $A\cong A\oplus A$. Being bases, they are certainly maximal linearly independent sets.

$\endgroup$
6
  • $\begingroup$ Thanks! I was considering commutative ring, though. $\endgroup$
    – ashpool
    Jun 30, 2010 at 16:33
  • 1
    $\begingroup$ Duh (to myself); you did tag it "commutative algebra." I believe there can be no examples in the commutative case, because commutative rings have IBN (invariant basis number). $\endgroup$ Jun 30, 2010 at 16:43
  • 1
    $\begingroup$ That rules out free modules (IBN means that the rank of a free module is well-defined), but not arbitrary modules. $\endgroup$ Jun 30, 2010 at 17:18
  • $\begingroup$ @Arturo: are you sure? So far as I know, the property IBN pertains to free modules only. (But maybe the arguments carry over to this context; I haven't thought it through.) $\endgroup$ Jun 30, 2010 at 17:21
  • $\begingroup$ No, I'm not sure; indeed, IBN applies only to free modules, which is why I only "believe" this to be the case in general for commutative rings. Intuitively, I'm thinking that if I had maximal l.i. sets $x_1,\ldots,x_n$ and $y_1,\ldots,y_m$, some nonzero multiple of each $y_i$ would lie in $\langle x_1,\ldots,x_n\rangle$ (which would be free), and vice-versa; and then one could try to move this completly into a free setting and invoke IBN. But I run into technical problems (e.g,I think it is not in general true that if $y_1,\ldots,y_m$ is l.i., and $a\neq 0$, then so is $ay_1,\ldots,ay_m$). $\endgroup$ Jun 30, 2010 at 17:32

Your Answer

By clicking “Post Your Answer”, you agree to our terms of service and acknowledge you have read our privacy policy.

Not the answer you're looking for? Browse other questions tagged or ask your own question.